dayme11
Thanks Received: 0
Forum Guests
 
Posts: 8
Joined: June 23rd, 2010
 
 
 

Q5 - Television executives recently announced

by dayme11 Wed Sep 22, 2010 3:20 pm

Please explain why the answer is E instead of D. Thanks!
User avatar
 
bbirdwell
Thanks Received: 864
Atticus Finch
Atticus Finch
 
Posts: 803
Joined: April 16th, 2009
 
 
 

Re: Q5 - Television executives recently announced

by bbirdwell Thu Sep 23, 2010 1:35 pm

We have this conclusion: Television advertising will not be harder to sell next year, despite rising costs for those ads.

And this evidence:
Advertisers will continue to profit.

We are asked to strengthen the argument, so we need to provide a reason for the advertisers to continue buying ads despite the significant increase in cost. We have to make sure that the rising costs don't outweigh the continuing profits.

(E) does this by showing that people are watching more and more television. This fact supports the idea that advertisers will continue to buy ads.

As for the other answers:

(A) is about the cost of the products--out of scope. If anything, this makes it less likely companies can afford the pricier ads!

(B) is out of scope. Size rating system?

(C) might actually weaken the argument--if companies can no longer purchase the smaller chunks of advertising, they may be even less able to afford ads.

(D) does not help at all. It simply splits the field of advertisers in two and says that one is buying more and the other less. Without more data, we cannot know how this affects the actual numbers. Perhaps far more advertisers fall into the category that is decreasing. This would actually weaken the argument.
I host free online workshop/Q&A sessions called Zen and the Art of LSAT. You can find upcoming dates here: http://www.manhattanlsat.com/zen-and-the-art.cfm
User avatar
 
LSAT-Chang
Thanks Received: 38
Atticus Finch
Atticus Finch
 
Posts: 479
Joined: June 03rd, 2011
 
 
trophy
Most Thankful
trophy
First Responder
 

Re: Q5 -

by LSAT-Chang Sun Aug 14, 2011 4:24 pm

Would answer choice (C) not strengthen the argument because we don't know how long advertising time was this fall? At first I was tempted by this answer because I thought: "hmm 30 seconds does sound very little -- so if advertisers were given more than 30 seconds for their purchase than they would be more likely to buy those times" but that might have been an additional assumption I made on my side since advertisers could have had more than 30 seconds this fall but had paid less for it than what they would have to pay next fall. Am I correct?
 
zee.brad
Thanks Received: 3
Forum Guests
 
Posts: 28
Joined: February 02nd, 2012
 
 
 

Re: Q5 -

by zee.brad Sat Feb 04, 2012 9:27 am

changsoyeon Wrote:Would answer choice (C) not strengthen the argument because we don't know how long advertising time was this fall? At first I was tempted by this answer because I thought: "hmm 30 seconds does sound very little -- so if advertisers were given more than 30 seconds for their purchase than they would be more likely to buy those times" but that might have been an additional assumption I made on my side since advertisers could have had more than 30 seconds this fall but had paid less for it than what they would have to pay next fall. Am I correct?


Okay, here is my thought regarding answer choice C. We don't know for sure if they would like to buy ads longer than 30 secs or not, and we even don't know if the 30secs threshold sounds or not, so C doesn't make a good choice comparing with E, which we know for sure, more people watching, more potential customers, and more profit they will potentially earn, so the bigger chance they will continue to buy those ads time.
User avatar
 
bbirdwell
Thanks Received: 864
Atticus Finch
Atticus Finch
 
Posts: 803
Joined: April 16th, 2009
 
 
 

Re: Q5 -

by bbirdwell Tue Feb 14, 2012 6:01 pm

changsoyeon Wrote:that might have been an additional assumption I made on my side


Yep. So easy to do this when evaluating answer choices. When in doubt, err on the side of sticking closely to what is mentioned in the argument.

Here, the length of actual commercials is not mentioned, so any inclusion of that concept into our reasoning will require additional assumptions. As zee.brad pointed out, we have no idea what the significance of the 30 second minimum will be; perhaps zero blocks less than 30 seconds were purchased last year anyway -- without more data (or assumptions), we can't know.
I host free online workshop/Q&A sessions called Zen and the Art of LSAT. You can find upcoming dates here: http://www.manhattanlsat.com/zen-and-the-art.cfm
 
wj097
Thanks Received: 0
Atticus Finch
Atticus Finch
 
Posts: 123
Joined: September 10th, 2012
 
 
 

Re: Q5 - Television executives recently announced

by wj097 Mon Apr 08, 2013 10:33 pm

bbirdwell Wrote:(E) does this by showing that people are watching more and more television. This fact supports the idea that advertisers will continue to buy ads.


I have 2 interpretation as to why (E) is correct, but not sure which one is the proper approach.
1. It strengthens the conclusion on an separate independent base regardless of the premise.
2. It strengthens the argument by defending from possible situation that undermines the necessary assumption ("other factor has not changed significantly to outweigh the continued profitable TV ad selling environment")

Thx
User avatar
 
noah
Thanks Received: 1192
Atticus Finch
Atticus Finch
 
Posts: 1541
Joined: February 11th, 2009
 
This post thanked 1 time.
 
 

Re: Q5 - Television executives recently announced

by noah Tue Apr 09, 2013 7:33 pm

wj097 Wrote:I have 2 interpretation as to why (E) is correct, but not sure which one is the proper approach.
1. It strengthens the conclusion on an separate independent base regardless of the premise.
2. It strengthens the argument by defending from possible situation that undermines the necessary assumption ("other factor has not changed significantly to outweigh the continued profitable TV ad selling environment")
Thx

It's rare that a strengthen/weaken answer will go after the conclusion instead of the argument. I see this fitting into the majority. The argument's assumption, as you reference, is that there isn't some other factor that makes the continued profits outweighed by the rising costs. (E) tells us that indeed, the outweighing has not happened.

Thinking aloud here: does (E) really support the conclusion very well? I don't think so. Just because viewership is going up can we say it'll be no harder to sell ads next fall? That's a pretty weak argument since there are so many reasons ads could be harder to sell regardless of rising viewership.
 
cvfh17
Thanks Received: 0
Forum Guests
 
Posts: 22
Joined: March 30th, 2013
 
 
 

Re: Q5 - Television executives recently announced

by cvfh17 Sun Apr 21, 2013 4:47 pm

I know that E sounds better than D, The conclusion says that the advertisement time will be no harder to sell, but D says that advertisement time pruchased is increasing, and time for ad purchased is decreasing. so the fact that are purchasing less time on the ad doesn't mean that are not purchasing television advertisemt in general. It's not harder to sell.